Proving Propositions: A Challenge!

  • Thread starter Thread starter guroten
  • Start date Start date
  • Tags Tags
    Challenge
Click For Summary
The discussion centers on proving three mathematical propositions related to rational and irrational numbers, specifically involving the square root of 2. The first proposition, stating that if x is rational, then x + sqrt(2) is irrational, is confirmed to be true. However, the second and third propositions are challenged, with participants seeking counterexamples to demonstrate their falsehood. Suggestions for counterexamples include expressions like e(sqrt(2)) and sqrt(3)*sqrt(2), though there is uncertainty about their rationality. The conversation highlights the complexity of these propositions and the difficulty in finding definitive proofs or counterexamples.
guroten
Messages
32
Reaction score
0

Homework Statement


Prove these propositions


Homework Equations


If x is rational, then x + sqrt(2) is irrational.
If x + sqrt(2) is irrational, then x is rational.
If xsqrt(2) is irrational, then x is rational.

The Attempt at a Solution


I've tried a couple of ways, but I always end up at a dead end. usually, I end up assuming what I need to prove.
 
Physics news on Phys.org
guroten said:

Homework Statement


Prove these propositions


Homework Equations


If x is rational, then x + sqrt(2) is irrational.
If x + sqrt(2) is irrational, then x is rational.
If xsqrt(2) is irrational, then x is rational.

The Attempt at a Solution


I've tried a couple of ways, but I always end up at a dead end. usually, I end up assuming what I need to prove.

The first proposition is true. The others aren't. Can you find a counterexample for them? As to proving the first can you show us what you tried?
 
After working with it some more, I've gotten all but the last. Is a counter example something like e(sqrt(2))? But I don't know if that is rational or not.
 
guroten said:
After working with it some more, I've gotten all but the last. Is a counter example something like e(sqrt(2))? But I don't know if that is rational or not.

That's too hard. In fact, I'm not sure anybody knows the answer to that. How about sqrt(3)*sqrt(2)?
 
Question: A clock's minute hand has length 4 and its hour hand has length 3. What is the distance between the tips at the moment when it is increasing most rapidly?(Putnam Exam Question) Answer: Making assumption that both the hands moves at constant angular velocities, the answer is ## \sqrt{7} .## But don't you think this assumption is somewhat doubtful and wrong?

Similar threads

  • · Replies 8 ·
Replies
8
Views
2K
Replies
7
Views
2K
Replies
8
Views
2K
  • · Replies 4 ·
Replies
4
Views
5K
  • · Replies 8 ·
Replies
8
Views
2K
  • · Replies 8 ·
Replies
8
Views
1K
  • · Replies 2 ·
Replies
2
Views
1K
Replies
10
Views
2K
Replies
10
Views
2K
  • · Replies 4 ·
Replies
4
Views
2K